n-th order polynomial with all roots where all coefficients are 1 or -1, highest order of n?












7












$begingroup$


I have polynomial $f(x) = sum_{i=0}^n a_i x^i $, where $a_i = pm 1$. All roots of $f(x)$ are real. What's the highest order of $n$? Note that the roots are real but can be irrational. Even if there are duplicate roots, that's fine










share|cite|improve this question











$endgroup$












  • $begingroup$
    I assume you are not counting multiplicity of the roots (i.e. the n roots are all distinct).
    $endgroup$
    – D.B.
    Dec 25 '18 at 4:02










  • $begingroup$
    @D.B. yes, even if there are duplicate roots, that's fine
    $endgroup$
    – CodeNoob
    Dec 25 '18 at 4:04






  • 1




    $begingroup$
    An example with $n=3$ is $x^3-x^2-x+1 = (x-1)^2(x+1)$. There don't seem to be any for $n=4, 5$, $6$ or $7$.
    $endgroup$
    – Robert Israel
    Dec 25 '18 at 4:15












  • $begingroup$
    @RobertIsrael that's right. We can prove all roots have abs value between (0.5,2) as well.
    $endgroup$
    – CodeNoob
    Dec 25 '18 at 4:40










  • $begingroup$
    Descartes’ Rule of Signs might be helpful here.
    $endgroup$
    – Clayton
    Dec 25 '18 at 4:49
















7












$begingroup$


I have polynomial $f(x) = sum_{i=0}^n a_i x^i $, where $a_i = pm 1$. All roots of $f(x)$ are real. What's the highest order of $n$? Note that the roots are real but can be irrational. Even if there are duplicate roots, that's fine










share|cite|improve this question











$endgroup$












  • $begingroup$
    I assume you are not counting multiplicity of the roots (i.e. the n roots are all distinct).
    $endgroup$
    – D.B.
    Dec 25 '18 at 4:02










  • $begingroup$
    @D.B. yes, even if there are duplicate roots, that's fine
    $endgroup$
    – CodeNoob
    Dec 25 '18 at 4:04






  • 1




    $begingroup$
    An example with $n=3$ is $x^3-x^2-x+1 = (x-1)^2(x+1)$. There don't seem to be any for $n=4, 5$, $6$ or $7$.
    $endgroup$
    – Robert Israel
    Dec 25 '18 at 4:15












  • $begingroup$
    @RobertIsrael that's right. We can prove all roots have abs value between (0.5,2) as well.
    $endgroup$
    – CodeNoob
    Dec 25 '18 at 4:40










  • $begingroup$
    Descartes’ Rule of Signs might be helpful here.
    $endgroup$
    – Clayton
    Dec 25 '18 at 4:49














7












7








7


2



$begingroup$


I have polynomial $f(x) = sum_{i=0}^n a_i x^i $, where $a_i = pm 1$. All roots of $f(x)$ are real. What's the highest order of $n$? Note that the roots are real but can be irrational. Even if there are duplicate roots, that's fine










share|cite|improve this question











$endgroup$




I have polynomial $f(x) = sum_{i=0}^n a_i x^i $, where $a_i = pm 1$. All roots of $f(x)$ are real. What's the highest order of $n$? Note that the roots are real but can be irrational. Even if there are duplicate roots, that's fine







number-theory polynomials






share|cite|improve this question















share|cite|improve this question













share|cite|improve this question




share|cite|improve this question








edited Dec 25 '18 at 14:37







CodeNoob

















asked Dec 25 '18 at 3:45









CodeNoobCodeNoob

23019




23019












  • $begingroup$
    I assume you are not counting multiplicity of the roots (i.e. the n roots are all distinct).
    $endgroup$
    – D.B.
    Dec 25 '18 at 4:02










  • $begingroup$
    @D.B. yes, even if there are duplicate roots, that's fine
    $endgroup$
    – CodeNoob
    Dec 25 '18 at 4:04






  • 1




    $begingroup$
    An example with $n=3$ is $x^3-x^2-x+1 = (x-1)^2(x+1)$. There don't seem to be any for $n=4, 5$, $6$ or $7$.
    $endgroup$
    – Robert Israel
    Dec 25 '18 at 4:15












  • $begingroup$
    @RobertIsrael that's right. We can prove all roots have abs value between (0.5,2) as well.
    $endgroup$
    – CodeNoob
    Dec 25 '18 at 4:40










  • $begingroup$
    Descartes’ Rule of Signs might be helpful here.
    $endgroup$
    – Clayton
    Dec 25 '18 at 4:49


















  • $begingroup$
    I assume you are not counting multiplicity of the roots (i.e. the n roots are all distinct).
    $endgroup$
    – D.B.
    Dec 25 '18 at 4:02










  • $begingroup$
    @D.B. yes, even if there are duplicate roots, that's fine
    $endgroup$
    – CodeNoob
    Dec 25 '18 at 4:04






  • 1




    $begingroup$
    An example with $n=3$ is $x^3-x^2-x+1 = (x-1)^2(x+1)$. There don't seem to be any for $n=4, 5$, $6$ or $7$.
    $endgroup$
    – Robert Israel
    Dec 25 '18 at 4:15












  • $begingroup$
    @RobertIsrael that's right. We can prove all roots have abs value between (0.5,2) as well.
    $endgroup$
    – CodeNoob
    Dec 25 '18 at 4:40










  • $begingroup$
    Descartes’ Rule of Signs might be helpful here.
    $endgroup$
    – Clayton
    Dec 25 '18 at 4:49
















$begingroup$
I assume you are not counting multiplicity of the roots (i.e. the n roots are all distinct).
$endgroup$
– D.B.
Dec 25 '18 at 4:02




$begingroup$
I assume you are not counting multiplicity of the roots (i.e. the n roots are all distinct).
$endgroup$
– D.B.
Dec 25 '18 at 4:02












$begingroup$
@D.B. yes, even if there are duplicate roots, that's fine
$endgroup$
– CodeNoob
Dec 25 '18 at 4:04




$begingroup$
@D.B. yes, even if there are duplicate roots, that's fine
$endgroup$
– CodeNoob
Dec 25 '18 at 4:04




1




1




$begingroup$
An example with $n=3$ is $x^3-x^2-x+1 = (x-1)^2(x+1)$. There don't seem to be any for $n=4, 5$, $6$ or $7$.
$endgroup$
– Robert Israel
Dec 25 '18 at 4:15






$begingroup$
An example with $n=3$ is $x^3-x^2-x+1 = (x-1)^2(x+1)$. There don't seem to be any for $n=4, 5$, $6$ or $7$.
$endgroup$
– Robert Israel
Dec 25 '18 at 4:15














$begingroup$
@RobertIsrael that's right. We can prove all roots have abs value between (0.5,2) as well.
$endgroup$
– CodeNoob
Dec 25 '18 at 4:40




$begingroup$
@RobertIsrael that's right. We can prove all roots have abs value between (0.5,2) as well.
$endgroup$
– CodeNoob
Dec 25 '18 at 4:40












$begingroup$
Descartes’ Rule of Signs might be helpful here.
$endgroup$
– Clayton
Dec 25 '18 at 4:49




$begingroup$
Descartes’ Rule of Signs might be helpful here.
$endgroup$
– Clayton
Dec 25 '18 at 4:49










1 Answer
1






active

oldest

votes


















8












$begingroup$

Vieta's Formulas are the key to this problem. Let $r_1, cdots, r_n$ be the roots. Then define
begin{align}
A &= sum_{i=1}^n r_i \
B &= sum_{1 le i_2 < i_2 le n} r_{i_1}r_{i_2} \
C &= prod_{i=1}^n r_i .
end{align}

By Vieta's Formulas, we know that $A, B, C in {pm 1}.$ Now
$$sum_{i=1}^n r_i^2 = A^2 - 2B = 1-2B ge 0 implies B = -1.$$
But then by AM-GM, we have
$$frac{3}n = frac{1}n sum_{i=1}^n r_i^2 ge left(C^2 right)^{1/n} = 1 $$
which cannot happen for $n ge 4$.






share|cite|improve this answer









$endgroup$














    Your Answer








    StackExchange.ready(function() {
    var channelOptions = {
    tags: "".split(" "),
    id: "69"
    };
    initTagRenderer("".split(" "), "".split(" "), channelOptions);

    StackExchange.using("externalEditor", function() {
    // Have to fire editor after snippets, if snippets enabled
    if (StackExchange.settings.snippets.snippetsEnabled) {
    StackExchange.using("snippets", function() {
    createEditor();
    });
    }
    else {
    createEditor();
    }
    });

    function createEditor() {
    StackExchange.prepareEditor({
    heartbeatType: 'answer',
    autoActivateHeartbeat: false,
    convertImagesToLinks: true,
    noModals: true,
    showLowRepImageUploadWarning: true,
    reputationToPostImages: 10,
    bindNavPrevention: true,
    postfix: "",
    imageUploader: {
    brandingHtml: "Powered by u003ca class="icon-imgur-white" href="https://imgur.com/"u003eu003c/au003e",
    contentPolicyHtml: "User contributions licensed under u003ca href="https://creativecommons.org/licenses/by-sa/3.0/"u003ecc by-sa 3.0 with attribution requiredu003c/au003e u003ca href="https://stackoverflow.com/legal/content-policy"u003e(content policy)u003c/au003e",
    allowUrls: true
    },
    noCode: true, onDemand: true,
    discardSelector: ".discard-answer"
    ,immediatelyShowMarkdownHelp:true
    });


    }
    });














    draft saved

    draft discarded


















    StackExchange.ready(
    function () {
    StackExchange.openid.initPostLogin('.new-post-login', 'https%3a%2f%2fmath.stackexchange.com%2fquestions%2f3051827%2fn-th-order-polynomial-with-all-roots-where-all-coefficients-are-1-or-1-highest%23new-answer', 'question_page');
    }
    );

    Post as a guest















    Required, but never shown

























    1 Answer
    1






    active

    oldest

    votes








    1 Answer
    1






    active

    oldest

    votes









    active

    oldest

    votes






    active

    oldest

    votes









    8












    $begingroup$

    Vieta's Formulas are the key to this problem. Let $r_1, cdots, r_n$ be the roots. Then define
    begin{align}
    A &= sum_{i=1}^n r_i \
    B &= sum_{1 le i_2 < i_2 le n} r_{i_1}r_{i_2} \
    C &= prod_{i=1}^n r_i .
    end{align}

    By Vieta's Formulas, we know that $A, B, C in {pm 1}.$ Now
    $$sum_{i=1}^n r_i^2 = A^2 - 2B = 1-2B ge 0 implies B = -1.$$
    But then by AM-GM, we have
    $$frac{3}n = frac{1}n sum_{i=1}^n r_i^2 ge left(C^2 right)^{1/n} = 1 $$
    which cannot happen for $n ge 4$.






    share|cite|improve this answer









    $endgroup$


















      8












      $begingroup$

      Vieta's Formulas are the key to this problem. Let $r_1, cdots, r_n$ be the roots. Then define
      begin{align}
      A &= sum_{i=1}^n r_i \
      B &= sum_{1 le i_2 < i_2 le n} r_{i_1}r_{i_2} \
      C &= prod_{i=1}^n r_i .
      end{align}

      By Vieta's Formulas, we know that $A, B, C in {pm 1}.$ Now
      $$sum_{i=1}^n r_i^2 = A^2 - 2B = 1-2B ge 0 implies B = -1.$$
      But then by AM-GM, we have
      $$frac{3}n = frac{1}n sum_{i=1}^n r_i^2 ge left(C^2 right)^{1/n} = 1 $$
      which cannot happen for $n ge 4$.






      share|cite|improve this answer









      $endgroup$
















        8












        8








        8





        $begingroup$

        Vieta's Formulas are the key to this problem. Let $r_1, cdots, r_n$ be the roots. Then define
        begin{align}
        A &= sum_{i=1}^n r_i \
        B &= sum_{1 le i_2 < i_2 le n} r_{i_1}r_{i_2} \
        C &= prod_{i=1}^n r_i .
        end{align}

        By Vieta's Formulas, we know that $A, B, C in {pm 1}.$ Now
        $$sum_{i=1}^n r_i^2 = A^2 - 2B = 1-2B ge 0 implies B = -1.$$
        But then by AM-GM, we have
        $$frac{3}n = frac{1}n sum_{i=1}^n r_i^2 ge left(C^2 right)^{1/n} = 1 $$
        which cannot happen for $n ge 4$.






        share|cite|improve this answer









        $endgroup$



        Vieta's Formulas are the key to this problem. Let $r_1, cdots, r_n$ be the roots. Then define
        begin{align}
        A &= sum_{i=1}^n r_i \
        B &= sum_{1 le i_2 < i_2 le n} r_{i_1}r_{i_2} \
        C &= prod_{i=1}^n r_i .
        end{align}

        By Vieta's Formulas, we know that $A, B, C in {pm 1}.$ Now
        $$sum_{i=1}^n r_i^2 = A^2 - 2B = 1-2B ge 0 implies B = -1.$$
        But then by AM-GM, we have
        $$frac{3}n = frac{1}n sum_{i=1}^n r_i^2 ge left(C^2 right)^{1/n} = 1 $$
        which cannot happen for $n ge 4$.







        share|cite|improve this answer












        share|cite|improve this answer



        share|cite|improve this answer










        answered Dec 25 '18 at 5:15









        Sandeep SilwalSandeep Silwal

        5,91811337




        5,91811337






























            draft saved

            draft discarded




















































            Thanks for contributing an answer to Mathematics Stack Exchange!


            • Please be sure to answer the question. Provide details and share your research!

            But avoid



            • Asking for help, clarification, or responding to other answers.

            • Making statements based on opinion; back them up with references or personal experience.


            Use MathJax to format equations. MathJax reference.


            To learn more, see our tips on writing great answers.




            draft saved


            draft discarded














            StackExchange.ready(
            function () {
            StackExchange.openid.initPostLogin('.new-post-login', 'https%3a%2f%2fmath.stackexchange.com%2fquestions%2f3051827%2fn-th-order-polynomial-with-all-roots-where-all-coefficients-are-1-or-1-highest%23new-answer', 'question_page');
            }
            );

            Post as a guest















            Required, but never shown





















































            Required, but never shown














            Required, but never shown












            Required, but never shown







            Required, but never shown

































            Required, but never shown














            Required, but never shown












            Required, but never shown







            Required, but never shown







            Popular posts from this blog

            Plaza Victoria

            In PowerPoint, is there a keyboard shortcut for bulleted / numbered list?

            How to put 3 figures in Latex with 2 figures side by side and 1 below these side by side images but in...